What’s the right answer and why

Fast-food restaurants make up 45 percent of all restaurants in Cantaria. Customers at these restaurants tend to be young; in fact, studies have shown that the older people get, the less likely they are to eat in fast-food restaurants. Since the average age of the Canatrian population is gradually rising and will continue to do so, the number of fast-food restaurants is likely to decrease.

Which of the following, if true, most seriously weakens the argument?

A) Fast-food restaurants in Canatria are getting bigger, so each one can serve more customers.

B) Some older people eat at fast-food restaurants more frequently than the average young person.

C) Manypeoplewhorarelyeatinfast-foodrestaurantsneverthelesseat regularly in restaurants.

D) The overall population of Canatria is growing steadily.

E) As the population of Canatria gets older, more people are eating at

home.

Beginner Asked on January 5, 2017 in Critical Reasoning.
Add Comment
3 Answer(s)

D is the correct answer.

You are asked to weaken the conclusion – “the number of fast-food restaurants is likely to decrease.”

If the population is growing, that means the number of young people will also increase. Then, we cannot say with certainty that the number of restaurants will reduce.

 

Expert Answered on January 6, 2017.
Add Comment

Thanks, so I had also marked D but I was unable to figure out why B is incorrect.

Beginner Answered on January 6, 2017.
Add Comment

the key here is “some”.

You are making an inference based on outliers. (you already know what the general trend is – “the older people get, the less likely they are to eat in fast-food restaurants.”)

Expert Answered on January 6, 2017.
Add Comment

Your Answer

By posting your answer, you agree to the privacy policy and terms of service.